LSAT and Law School Admissions Forum

Get expert LSAT preparation and law school admissions advice from PowerScore Test Preparation.

General questions relating to the LSAT Logic Games.
User avatar
 ec623ec623
  • Posts: 6
  • Joined: Mar 21, 2023
|
#100669
If there's a rule like 'G in 3rd only if Q is 5th' - I would diagram the rule as G3 -> Q5.

If I'm not mistaken, then doing the contrapositive it would be /Q5-> /G3. Does this mean if Q is not 5th, then G cannot be 3rd? Or just that G does not necessarily need to be 3rd (but is still able to be 3rd without Q being 5th)
User avatar
 Dave Killoran
PowerScore Staff
  • PowerScore Staff
  • Posts: 5853
  • Joined: Mar 25, 2011
|
#100673
Hi EC,

Yes, well done, that's exactly what it means! The G3 -> Q5 relationship is incredibly powerful, and via the contrapositive, when Q is not in 5, then G has no chance of being in 3. That forces G into another spot aside from 3.

So, it's not that G is always in 3, but when it is, Q MUST be in 5.

Please let me know if that helps. Thanks!

Get the most out of your LSAT Prep Plus subscription.

Analyze and track your performance with our Testing and Analytics Package.